Thursday 31 May 2018

CBSE CLASS 9 IMPORTANT QUESTIONS OF BIOLOGY


                        Cell- A basic unit of life                  

1. Name:
(a )The process through which amoeba acquires its food from the external environment .
(b) Name cell organelles that is termed as suicidal bags of the cell and has membrane bound sac filled with powerful digestive enzymes.
(c)Two cell organelles that contain their own genetic material .
(d) Site of energy release inside the cell.
(e) The main constituent substance present in the plant cell wall.
(f) The kind of plastid which is important for photosynthesis in leaves of the plants.
(g) The cell organelle that detoxifies poisons and drugs into liver of vertebrates. 
(h) Two cell organelles having double membrane envelope.
(i) Two materials stored in leucoplasts.
(j) The organelles which is associated with protein synthesis.
(k) Two components of chromosomes.
(l) The cell which have changing shape.
(m) The cell which have a typical shape.
(n) The plastid involved in conversion of green brinjal to violet .
(o) The power house of the cell.
(p) The cell organelle responsible for inter cellular transport.
(q) The functional unit of gene.
(r) A cell organelle which lacks membrane. Two nucleic acids present in the cell.
 2. State the function of chromosomes in a cell.
3.What is plasmolysis ?What happens to a plasmolysed cell when it is placed in water
4.What is plasma membrane? What are its functions?
5. What are cell organelles?
6.What is cellulose and its function?
7. How are vacuoles in plant cells different from those in animal cells
8. Why is it said that "a cell without nucleus is without any feature"?
9. What do you mean by nucleoid?
10. What is endoplasmic reticulum? Name the two types of endoplasmic reticulum. W.rite its main function.
11. What is a centrosome and what function does it perform?
12. Write the function of Chromatin material!
13. Describe in short that chief components of a nucleus.
14. Why are mitochondria also referred to as strange organelles?
15. What role does a nucleus play in cellular reproduction?
16. Distinguish between cell wall and cell membrane.
17. Give difference between cytoplasm and nucleus.
18. Write the main functions of cell wall.
19. Give any two functions of plastids.
20. Give any two functions of plasma membrane.
21.Name two nucleic acids present in the cell and their function.
22. Define osmosis. In what two ways it is different from diffusion?
23. Write one function each of ribosomes , vacuole,  plasma membrane.
24. Write two similarities and one of the dissimilarities between mitochondria and plastid.
25. Division of labor exists even at intracellular level. Justify the statement.
26. (a) L.ist any two structural differences and two similarities between a plant cell and animal cell (b) What would happen if an animal cell is kept in distilled water for 24 hours?
27. (a) Write two points of difference between nuclear region of a bacterial cell and a region of an animal cell.
(b) Which  structure present in the nucleus of a living cell bear genes?
28. Draw a diagram of a plant cell and label its any for parts.
29. (a) Level the part marked of 1, 2, 3 in a prokaryotic cell. (b)Mention any three feature of prokaryotic cells.
30. Differentiate between rough and smooth endoplasmic reticulum. How is endoplasmic reticulum inportant for membrane biogenesis?
31. Describe the role played by the lysosomes in a Cell .Why these are termed as suicial bags? How do they perform their function?
32. Distinguish leucoplast from chromoplast based on one feature. Give an example of chromoplast present in plant cell .What is the function of chromoplast in the plant cell?
33. Why is plasma membrane referred to as selectively permeable membrane? Give two points of difference between diffusion and osmosis.
34. Who give their term Golgi apparatus? Name one cell organelles that is formed by Golgi apparatus. .Write any two functions of Golgi apparatus.
35. Endocytosis is found only in animals. State and explain the process by which an amoeba obtain its food.
36.Draw and label the parts of prokaryotic cell.
37. Write the functions of the following organelles in a cell (i) chloroplast (ii) nucleus (iii) ribosomes
38.Two beakers A&B contain plain water and concentrated sugar solution respectively equal number of dry raisins are kept in them for a few hours and then taken out.
Explain the reason for the difference in the physical appearance of raisins which were taken out of the two beakers on the basis of above observation category is that two solutions as hypotonic and hypertonic.
39.Which cell organelles would you associate with ATP Productions ?How is this organization to make its own protein?
40. A student performed an experiment by placing the de-shelled egg .in a concentrated salt solution for 5 minutes. What changes did he observe in the egg? Give reason for the same.
41. (a)Why is the cell called the structural and functional unit of life? (b)Why is the plasma membrane called a selectively permeable membrane? (c)Name the factors which decides the movement of water across the plasma membrane.
42. State reasons for the following:
(a) Mentioned the use of deep folding in the inner membrane of mitochondria .
(b) Plastids are able to make their own protein .
(c) Plant cells shrink when kept in hypertonic solution.
43. Define osmosis? In what two ways it is different from diffusion?
44. Name three types of plastids found in plant cells and give one functions of each.
45. What is membrane biogenesis? Explain what happens when a drop of concentrated sugar solution is placed on a real Leaf peel mounted on a glass slide? Name this phenomenon. Would the same happen if the rear leaf was boiled before mounting? Give reason for your answer.
46.What would happen if the plasma membrane rupture or breaks down ? What would happen to the life of a cell if there were no Golgi apparatus?
47. Describe the structure of the plant cell.
48. What is the name given to the therad shaped structures in the nucleus? Why is it important?
49. Draw a diagram of the nucleus to show the given parts: (i)  nucleolus (ii) nuclear pore (iii) nuclear envelope,
50. Explain the term (i) endocytosis (ii) plasmolysis .What will happen if the organisation of a cell is damaged due to certain physical or chemical reasons? (iii) How do substances like CO2 and water move in and out of the cell?
51. Differentiate between chromatin and chromosomes.
52. List any two single-celled organisms.                     
53. M.ention the difference between prokaryotes and eukaryoted in terms of nuclear reason present in them.

                                                                 TISSUES

1. State two functions of the adipose tissue
2.What are the types of complex permanent tissues?
3.What is a tissue?Justify that blood is a tissue
4. Name the two types of plant tissue.
5.What is meristmatic tissue?
6. How many types of meristmatics tissue are there on the basis of their presence?
7. What do you understand by differentiation?
8. How is permanent tissue formed from meristematic tissue?
9. What is source of cork?
10. What are stomata?
11. What do you mean by guard cell?
12. Name the complex plant tissue
13. What is the function of cutin the waxy substance present in epidermis of desert plants ?
14. What are vascular bundle?
15. Name the four elements of phloem.
16. What is the function of lymph?
17. Identify the meristematic tissue ,which are located at (i)growing tips of roots and stems(ii) the base of the leaves or internodes on twings.
18. What is apical meristem ? What is its function
19. Why is epidermis present as a thick waxy coating of cutin in desert plant?
20. Draw a level diagram of a neuron.
21. State the role of ligament and tendon in our skeletal system.
22. A horse and mango tree are both complex living organisms with the specialist yet different tissues system to perform the basic life processes. give two reasons for possessing different tissues to perform a similar function.
23. (i) Voluntary muscles are also known as skeletal muscles.Justify.
(ii)Give two structural characteristics of this voluntary muscles?
24.How are simple epithelial tissues and compound epithelial tissue different?
25.  Which epithelial tissue is present on the tongue?
26.Which two cells contains flat squamous cells arranged in many layers to prevent wear and tear of parts?
27.Name an animal whose the skeleton is made up of cartilage.
28. What is the function of RBCs?
29. What is the lifespan of human RBCs?
30.Write the location and function of collenchyma tissue.
31.State three advantages of cutin. .
32. Write any two characteristic feature of parenchyma tissue.
33. List two characteristics of call name the chemical present in them and mention its role
34. With the help of level diagram, differentiate parenchyma and collenchyma.
35.List any two characteristic feature of parenchyma tissue.
36.List any two functions of epithelial tissue in human body
37.Name the living components common to both the complex permanente tissues found in plants.
What is its function .
38.Name the tissue (a) that connect muscles to bone in human (b)forms inner lining of alveoli
(c) stores fat in our body (d) transports water and minerals in Plants
39. List four functions of blood
40. Write two locations of the following animal tissues (a) simple squamous epithelial cells
 (b) cuboidal epithelium.
41.(a) Name the tissue that makes husk of coconut.Write three characteristics of this tissue.
(b) Name the connective tissue which connects two bones.(c) Name the connective tissue present in external ear.
42.  Mention one reason in the human body where adipose tissues is present and state one function of the tissue.
43. List two points of differences between parenchyma and sclerenchyma .
44. Name the simple permanent tissue which (i) forms the basic packing tissue (ii) provides flexibility in plants.
45. Mention four characteristics feature of the cells of meristematic tissue.
46. What is apical meristem? Where it is located? State its function
47. How does the bone matrix differ from the matrix of cartilage?
48.Name the tissue which helps in transportation of oxygen that we inhale to various parts of our body. Write the composition of this tissue.
49. Water hyacinth plant floats on water surface. Name the tissue and its type due to which it is possible and also explain the special feature of this tissue that help in this.
50.Name the following tissue
(a) The connective tissue found between the skin and muscle
(b) The tissue which connects two bones
(c) The epithelial tissue which forms the lining of the kidney tubules
(d) The tissue which is present in the veins of leaves.
51. Growth in plant is restricted to certain regions. Give reason for this fact. Mention two growth in plants.
52. Identify the type of tissues in the following
(a) Vascular bundle (b) Inner lining of the intestine (c) Lining of kidney tubules (d) Iris of the eye
(e) Muscles of the heart (f) Bronchi of the lungs.
53. Draw a labelled diagram of the basic unit of nervous tissue .State the role of epidermis in plants.
54.State the difference between tendon and ligament.
55.(i) Name the following (a) tissue that connect muscles to Bone (b) tissues that store fat in our body (c) tissues that transport food in plants (d) tissues that provides flexibility in plants.
(ii)  List the role of cork in plants.
56. Label the following and give one function of each part labeled a , b ,c
57.Identify the animal faces from the given description and also mention their location in the human body.
 Tissue "A" cells are filled with a fat globules and the tissues act as an insulator
 Tissue "B" has cylindrical branched cells and the tissues shows the  rhythmic contraction and relaxation through life.
58. Write three distinguishes features between cells of meristematic and permanent plant tissue.
59. Mention three characteristics feature and three functions of xylem.
60. Explain in brief any three rolls of epidermis in plants.
61. Name any three connective tissue. Give any one function of each.
62. Explain how the bark of tree is formed? How does it act as a protective tissue .
63.Draw a diagram of cardiac muscle and level any two parts..Write one main function of cardiac muscle
64.Show using a diagram of meristematic tissues in plants on the basis of their location. I.dentify the one which is responsible for increase in the growth of stem.
65.State one point of difference between xylem and phloem. Draw a neat diagram of xylem vessel and tracheid.
66.Wight the difference between striated muscle, smooth muscle and cardiac muscles on the basis of their shape and the number of nuclei.
67. Make a labeled diagram to highlight to difference between striated and cardiac muscles .Write one function of striated muscles in our body .
68. What are the small pores observed in the epidermis of the leaf called .Write it's two main functions.
69. Give reasons for the following:
(a) bark of a tree is impervious two gases and water (b) in desert plants epidermis has a thick waxy coating(c) epidermal cells of the root generally have hair like parts.
70.Identify the simple permanent plant tissue with the following description and also mention their location in the plant body. (a) cells have irregular wall thickenings (b) tissues with large inter cellular space and cells having large air cavity (c) cells are long narrow and dead in nature.
71. Draw a labeled diagram of longitudinal section of sclerenchyma. Name any two reasons in the plant where this tissue is present.
72. Give reason for the following: (a)  Cells of sclerenchyma have an narrow lumen.
(b) Branches of a tree move and bend freely in high wind velocity. (c) I.t is difficult to pull out the husk of coconut .
73.Uma started walking fast when she noticed that some unknown faces are following her. Name the two types of tissues which facilitated the movement of her legs bones in response to the stimulus. 74.Draw the diagram of any one of the above mentioned two tissues and level any two parts.
75. Identify the type of muscular tissue having following characteristics
(a) Cylindrical branched and uni nucleated. (b) Long with pointed ends and uni nucleated (c) long cylindrical, unbranched and multi nucleated.
76. Name the type of epithelial tissues that lines the following :
(a) Oesophagus  (b) Respiratory tract  (c)  Kidney tubules  (d) Inner lining of intestine. (e) Blood vessels  (f) Ducts are saliva glands
77. Explain the process of differentiation. Identify the given tissue in the diagram given below. State any three characteristics feature of the same.
78. Name the type of tissue of blood and also the liquid matrix of the blood. Name the components present in it . List any two functions of blood.


                   BILOGICAL DIVERSITY

1.State two ways of classifying plants and animals into different categories.
2.(i) Name the plants having seeds with two cotyledons.
.(ii) Name the type of circulatory system in arthropods.
3.Write one point of difference each between the following:
(i) Porifera and Coelenterata
(ii) Birds and Reptiles.
4.List any three different features of whale and fish.
5.To which class of animalia does the housefly belong? Mention any three characteristics feature of this class of animal. Give two other examples of this class of animal.
6. Write any two differences between cryptogames and phaneroames.
7. (a) Given below arefew plant spieces,
Identified the division to which they belong and write one major characteristics of the division.
(i) Spyrogyra (ii) deodar (iii) moss.
(b) What is the mode of nutrition for all of them ?
8. The following is a list of invertebrates. Classified them into different phyla giving one characteristic morphological feature to justify your classification ?
(a) Starfish (b) Nereis
(c) Housefly (d) Sycon
 (e) Planaria.
9.Mention any two similarities and one difference in body design of animals belonging to Annelida and Arthropods.  Give two examples each of animals belonging to these two classes.
10.(a) Identify the organisms. Name the phylum to Which this organism belongs. Write any two characteristic feature of the phylum.
(b)Pick up the old one out and justify your choice by giving regions. Crocodile Salamander Sparrow Bat.
(c) Write the common name for a Ascaries ,Wuchereia.
11. Draw the flow chart for classification of Kingdom plantae right suitable examples of each group.
12.(a) Which organism is more complex and evilved among bacteria, mushroom and mango tree.Give reasons.
 (b)List out three common feature in cat, rat and bat.
13. (a) What is coelom. State its significance.
(b) pick of the organisms that have a pseudocoelom from the following:
Earthworm, pinworms, tape-worm and roundworms.
(c) What is peculiar about the column of arthropods? What is such a condition called? Explain.
(d) To which phylum of animals where do the following Animalia belong to
octopus, pila, chiton and unio? Comment on their coelom.
14. (a) Differentiate between fungi and plantae.
(b) Mention the basis of classification among plants to different levels.
(c) Differenc between Amphibians and Reptiles.
(d) Aves and Mammals.
15. Name the five classes of vertebrates,compare any two on the basis of their: (i)Habit (ii) Covering of skin (iii) Respiratory organs (iv) Chamber of heart (v)Reproduction.
16. (a)State two feature of Amphibians.
(b) Identify the following organisms:
(i) cold blooded animals that lay eggs in water and have three-chambered heart
(ii) spiny skinned organisms which have a peculiar water driven tube system that they use for moving around.
 (iii) Organisms which have an open circulatory system having blood filled coelomic cavity.
17.(a) State two characteristics features of vertebrates
 (b) State reasons for each of the following statements;
(i)  Ekdina and platypus lay egg but considered as mammals
(ii) Forelimbs of birds are modified
(iii) Crocodiles have four-chambered heart but are still reptiles.
(iv) Birds have pneumatic bones.
18. Why do we keep both snake and turtle in the same class?
19. Name the largest group of animals. Write the salient features of this group. Give two examples.
20. Differentiat between Bryophyta and Pteridophyta.
21. Classify following organisms on the based on presence of two coelom.
22. Draw diagram of hydra and label the following parts: Tentacles, Stinging cells, Gastrovascular cavity, Epidermis .
23. Write the four features of pteridophyta and give any two examples.
24. Write the names of the kingdom's for the following  organisms:
(a) Single celled , eukaryotic and photosynthetic
(b) Multicellular,  eukaryotic and photosynthetic
(c) Single celled, prokaryotic and heterotrophic.
25. "Tapeworm is triploblastic". What does it mean? How is hydra different from tapeworm in this aspect? Name the phylum to which hydra and tapeworm belong to?
26. Write one characteristics feature of each of the following:
(i) Cryptogam  (ii) Arthropoda  (iii) Mammals.
27. (i) What is the function of notochord?
 (ii) List out any four features that all chordates will possess.
29. Write three differences between animals belonging to Aves group and those in Mammals group .
30. (a) Write two distinct features by which vertebrates are differentiated from invertebrates.
(b) What are photochordates? Give two examples.
31. Write four characteristics of monerans.
Blue-green algae have been included under the Kingdom monera not under plantae. Why?
32. How are pteridophytes  different from phanerogames? Give example for each group.
33. You are given leech, Neries, Prawn and Scorpion and all have segmented body organizations. Give the important characters based on which you separate these organisms into different groups.
34. Write two examples of each:
(a) egg-laying mammals
(b) organisms with open circulatory system (c) prokaryotic organisms.
35. Write two examples of organisms belonging to the given phylum
(a) nematoda
(b) platyhelminthes
(c) arthropoda
36. What is the system of scientific naming of organisms called ? State two important conventions followed while giving such names.
37. State two reasons for the following:
(a) Fungi are called saprophytes.
(b) Platyhelminthes are called so.
(c) Birds have hollow bones
38. To which group in the hierarchy of classification saprophytes belong?  Why are they called saprophytes ? Ehat is meant by symbiotic relationship ? Give an example .
39. Distinguish between
(a) Monera and protista on the basis of nucleus
(b) Fungi and plantae on the basis of mode of nutrition.
(c) Gymnosperm and angiosperm on the basis of seeds.
40. Draw a neat labelled diagram of Spyrogyra and label the following parts:
(a) Outermost layer of the cell organelles that performs the function of photosynthesis
(b) jelly like substance in the cell where all organelles are suspended.
(c) Darkly colored and dot like structures generally present in the center of the cell.
40. Classify the following and write one characteristics of each
(a) Lichen  (b) Sponges  (c) Flatworm .
41. State reasons for the following:
(a) Protists have appendages like cilia and flagella.
(b) Angiosperm are called so
(c) Fish have scales on their body.
42. Name the phylum to which Spring skinned organisms like starfish and sea urchins belong to. Write two characteristics of this phylum which are peculiar of this group?
43. Write any three criteria on the basis of which organisms are grouped into five Kingdoms.
44. Write one difference between each of the following
(a) Echinodarmata and chordata.
(b) Amphibia and Pisces.
(c) Gymnosperm and Angiosperm.
45. Lichen show symbiotic life from between two organisms
(a) Name the two organisms showing this relationship
(b) Write the kingdoms to which each one of them belongs to.
(c) In such a relationship, organisms are parasitic or mutually benefited from each other or mutually dependent on another organism. "Explain".
46. Name the phylum to which the following organisms belong :
 (a) Organisms which have peculiar water driven tube system that they use for moving around.
(b) Organisms which have a foot that is used for moving around and have an open circulatory system.
(c) These organisms have holes or poles all over the body. These lead to canel system that helps in circulating water throughout the body.
47. Identify and name the following:
(a) Organisms that use dead and decaying organic organic material as food
(b) Cell walls of fungi are made up of this special type of sugar.
(c) Kingdom to which animal belongs
(d) An example of a moneran and an animal with the pseudocoelom
(e) A group which has an open circulatory system
48.(a) Draw a well labeled diagram of paramecium.(b) Name the kingdom to which it belongs.
49. Snails are soft bodies shelter animal. Identify the phylum to which it belongs . Write any two distinguishing features of this phylum.
49. Write two characteristic feature of phylum to which Ascaris belongs.Name the parasitic worm causing elephantiasis.
50. What is the difference between phylum platyhelminthes and nematoda?
51.(a)Name the Kingdom to which protozoa belong.(b) Name the appaendages presents for movement in the following organisms: )(i) Euglena (ii) Paramecium.(c) Write one importance characteristics of that kingdom.
52. Give other name of to the category of plants that are called phanerogams. How are they further classified on the basis of their seeds. Give example from each category.
53. How does an Angiosperm differ from a gymnosperm with respect to its reproductive structure. Give an example of organism belonging to each one of these two classes.
54. To which class do Salamander and sparrow belong ? Write any two difference between these classes.
55. What do you understand by the term'naked embryo'? Name any two divisions in kingdom 'Plantae' that have naked embryo.Write the example of each division
56. Name the scientist who introduced the binomial nomenclature .
(a) What is the scientific name of humans?
(b)  Find the odd one out : Riccia , Funaria, Fern, Marchantia.
57. State the appropriate term of the following:
(i) Plants which beer seeds with two cotyledons (ii) Animals which do not have coelom (iii)edible fungi.
58.(a) On what basis does the embryo of cryptogam differ from that of phanerogam? (b) Describe the feature that divides the angiosperms into two groups. (c) State that two subgroups of angiosperm.
59. List the basic criteria for classifying the plantae into various groups.Identify the following specimen given and write its characteristic features.
60. Give the reasons for the following
(i) Echinodna and platypus lay egg but are considered as mammals.
(ii) Crocodile has four-chambered heart but still a reptile.
(iii) Birds have pneumatic bones.

             HEALTH AND DISEASE

1.Influenza or common cold spreads faster and is difficult to control. Why?
2.State two different ways by which infectious diseases spread.
3.What is an antibiotic? Give one example.
4.Name the causal organism of AIDS. Why a person suffering from its, cannot fight even very minor infections?
5."Community Health" is essential for good individual health. Justify this statement giving examples.
6. With the help of an example explain how disease-causing microbes spread through air ?
7. Which of the following disease will cause major ill effects on General Health- Elephantasis ,Cough and Cold, Tuberculosis, and Diarrhea. What are such disease called ?
8. AIDS is a fatal disease, Explain why?
9. Health workers are exposed to more sick people than others in the community. Write any for preventive measures they take to avoid sickness.
10. How can we prevent water born and vector-borne infections ?
11. What are communicable disease ? write it's one example. How do diseases spread
12. What are vectors ? Give two examples ?
13.Write two examples of each:
(a) Viral disease   (b) Bacterial disease
14. Chances of spreading of cholera are higher in a village. Give reason.
15. Although Archana has been suffering from a cold and cough she decided to appear for her class test. Classmates seated close to her hand and exposure to the infection being carried by Archana. However only one of them actually suffered from cold and cough. Explain what prevented rest of those classmates catching cold and cough in spite of their exposure to the infection.
16. List for diseases for which vaccines have been developed.
17. (a) Name a worm which is found in our large intestine (b) Name the bacteria which can cause acne (c) Which one is responsible for sleeping sickness (d) Which disease is caused by the protozoan leishmania.
18.The immune system of a person is damaged by the attack of pathogen in his body.
(a) Name the disease he is suffering from (b) Name the pathogame and any to make modes of its transmission.
19. State any four ways by which and AIDS virus spread from an infected person to a healthy person.
20. State in tabular form the modes of transmission of each of the following disease:
(a) Syphilis   (b) Tuberculosis  (c) Jaundice  (d) Japanese encephalitis.
21. 'Being disease free is not the same as being healthy' Explain the statement given an example.
22. What causes encephalitis ? How does it enter the body? Which organ does it infect? What are the symptoms in this organ is infected?
23. (a) How do children in many parts of India get immune to Hepatitis 'A' by the time they are five year old?
(b) Pinky suffered from chicken pox last month. Her grandmother told her that now she would not suffer from this disease again . What is the reason for such a saying ?
24. What is immunization . List to disease against which vaccines are available
25.List four preventive measures against communicable diseases.
26. (a) What are vectors ? (b) In many species of mosquitoes the male do not prefer human blood ,but females do. State why?
27. Write any one biochemical  pathway in bacteria that is blocked by the antibiotics like penicillin.
28. Why is it difficult to make antiviral drugs?
29. Explain how individual health depends on social and mental well-being.
30. List any two ways of preventing the spread of Air borne diseases.
31. List any four essential factors that must be taken care of by an individual for keeping good health.
32. Making antiviral medicines is harder than making antibacterial medicine.Justify the statement.
33. Penicillin is not effective against common cold why?
34. Which disease is more harmful : acute or chronic disease ? Why are we advised to take bland and nourishing food when we are sick?
35. Write differences between acute and chronic diseases. Pick out chronic disease from the list given below: Japanese Encephalitis, viral fever, common cold, tuberculosis.
36. Write the symptoms when the following organs are targeted by microbes:
(a) Lungs (b) Liver (c) Brain
37. Same drug does not work against the microbes belonging to different groups. Why? State the mechanism of antibiotics in killing bacteria.
38. (a) State the principle of immunization. Name two diseases from which vaccine is available.
(b) Mention two basic principles of preventing of infectious diseases.
39. State the reasons for the following statements
(a) Children at the time of birth must be given proper vaccination (b) A person suffering from disease like tuberculosis , flu should be advised to avoid close public contact.(c) Personal hygiene is very essential for good health.
40. (a) While going abroad, why is it essential to get vaccinated against certain disease?
(b) Name of vaccine which saves the life of babies from three diseases. (c) A person is suffering from chest pain, breathlessness, loss of body weight, persistent cough and produces blood Stained sputum. Name the disease.
41. Write three precaution you take in your school to reduce the incidence of infectious disease.
42. (a) State the method of transmission of each of the following disease
(i) Cholera (ii) Malaria (iii) Pneumonia (iv) AIDS.
(b) Name the disease a person will get if the disease causing microbes target the liver of a person.
43. Differentiate between infectious and non-infectious disease. Give two examples of each.
44. Explain when a disease categories as a communicable disease. Give two examples for each of disease communicated through (i)  air and (ii) water.
45. (a) If penicillin is given to a person suffering from jaundice, it does not have any effect on the infection. Why?
(b) Name a disease which has been eradicated from the world (c) State the principle behind its eradication.
46. Antibiotics are successful in curing bacterial infections but do not cure viral infections. Why?
47. A man is suffering from AIDS. (i)  He is not able to fight off even minor infection. Why?
(ii) Write any two ways in which he could have got this disease.(iii) Will the treatment by antibiotics help him in AIDS ? Justify your answer.
48. For most microbes the organ they target is related to their point of entry.
(a)Furnish details of your answer under the following headings:
Organ of Entry   Kind of microbes   Target organ     Disease caused
Mouth                Bacteria
Mouth                Virus.
(b) State the two important ways by which an infectious disease can be treated.
49.(a)  State the principles of treatment that are generally followed by a doctor to treat infectious disease. (b)  Write two ways by which HIV may get transmitted from one person to the other.
50. State the reason for the following statements:
(a) AIDS is considered to be a syndrome and not a disease. (B) Antibiotics are ineffective against viral infections. (c) Our surroundings should be free from stagnant water.
51.(a) Which of these is an acute aliment and why? Tuberculosis, Cancer, Diarrhea, Elephantiasis.
 (b) State any two internal non-infectious causes of disease. (c) Name the organ that is targeted by the virus that causes jaundice.
52. List the name of three disease caused by virus stating their mode of communication is in each case.
53. Name two diseases caused by each of the following:
(a)  Bacteria       (b) Virus         (c) Fungi
54. (a) What is the basis of principle of immunization ?
(b) Why are majority of the children in many parts of India already immune to Hepatitis - A though. They have never been vaccinated against it?
55. How infectious diseases spread in a community? Write and give causes of diseases with example
56. Name any five different groups of infectious disease and give an example of a disease caused by each of them.
57. Explain giving reasons:
(a) Balanced diet is necessary for maintaining healthy body. (b) Health of an organisms depend upon the soundings environmental conditions. (c) Our surroundings area should be free of stagnant water. (d) Social harmony and good economic conditions are necessary for good health.
58. Why do we not use antibiotics against viral disease?
59. Why does the penicillin not effective against the human cell but effective against bacterial cell?
60. During HIV infection virus does not kill the person but other diseases killed them could you explain it?
61. Is the primary cause of marasmus? What is the main cause of infectious disease?
62. Which is the causative organisms of acne?
63. Why HIV is an incurable disease?
64 Who walk out the principal of immunization?
65. Name the causative organism.
66. Which disease is caused by Helicobacter pylori?
67. Why are signs of a disease more important than symptoms?
68. What is the major drawback of principle of treatment?
69. How do children in many parts of India get immune to Hepatitis- A by the time they are 5 years old.
70. Who discovered first vaccine and for which disease ?


                                           




                                              

Tuesday 29 May 2018

CLASS 12 PREVIOUS YEARS QUESTIONS OF CHEMISTRY

CBSE Class 12 Chemistry

General Instructions:
All questions are compulsory.
Questions number 1 to 5 are very short answer questions and carry 1 mark each.
Questions number 6 to 10 are short answer questions and carry 2 marks each.
Questions number 11 to 22 are also short answer questions and carry 3 marks each.
Question number 23 is a value based question and carry 4 marks.
Questions number 24 to 26 are long answer questions and carry 5 marks each.
Use log tables, if necessary. Use of calculators is not allowed.

                                                              1. Chemistry

1. What would be the nature of solid if there is no energy gap between valence band and
conduction band?
2. Which of the following reactions is SN1.type? =

3. Write the IUPAC name of the given compound:

4. On heating Copper turnings with conc. H2SO4 a colourless gas with pungent smell is evolved which decolourises acidified KMnO4 solution. Identify the gas with pungent smell.
5. Write the main reason for the stability of colloidal sols.
6. Show that the time required for completion of 3/4th of reaction of first order is twice that of half life (t1⁄2) of the reaction.
                                                                         OR
Derive integrated rate equation for rate constant of a zero order reaction.
7. When a coordination compound PtCl4.6NH3 is mixed with AgNO3, 4 moles of AgCl are
precipitated per mole of the compound? Write:
i. Structural formula of the complex ii. IUPAC name of the complex
8. Write the structures of the following molecules:
i. HS2O8        ii. XeF6
9. An organic compound ‘X’ having molecular formula C4H8O gives orange-red ppt. with 2,4-DNP reagent. It does not reduce tollen’s reagent but gives yellow ppt. of iodoform on heating with NaOI. Compound X on reduction with LiAIH4 gives compound 'y' which undergoes dehydration reaction on heating with concH2S04 to form But-2-ene. Identify the compounds X and Y.
10. Write two differences between a solution showing positive deviation and a solution
showing negative deviation from Raoult’s law.
11. a. For the complex [CoF6]3-, write the hybridization type, magnetic character and spin nature of the complex. (At.number : Co=27)     b. Why is the complex [Co(en)3]3+ more stable than the complex [CoF6]3-?
12. Define the following terms:
i. Peptization     ii. Zeta potential iii. Brownian movement
13. How do you convert:
i. Chlorobenzene to toluene ii. But-1-ene to But-2-ene iii. Ethanol to Ethyl iodide
                                                                   OR
What happens when:
i. n-butyl chloride is treated with alcoholic KOH.
ii. 2-chloropropane is treated with sodium in the presence of dry ether.
iii. Chlorobenzene is treated with CH3Cl in the presence of anhydrous AlCl3.
14. Write the chemical equations involved in the above reactions.Write the principle behind
the following:
i. Vapour phase refining ii. Chromatography iii. Froth floatation process
15. i. Write the name of mono saccharides which are obtained after the hydrolysis of Lactose.
ii. What type of bonding is responsible for the stability of -helix ?
iii. Write the difference between Nucleotide and Nucleoside.
16. Give reasons:
i. The a-hydrogen atoms of aldehydes and ketones are acidic in nature.
ii. Oxidation of aldehydes is easier than ketones.
iii. CH2=CH-COOH is more acidic than CH3-CH2COOH.
17. Give reasons:
i. Red phosphorus is less reactive than white phosphorus.
ii. Sulphur shows greater tendency for catenation than oxygen.
iii. ClF3 is known but FCl3 is not known.
18. Complete the following reactions:

19. For the first order thermal decomposition reaction, the following data were obtained:
20. Calculate the freezing point of a solution when 3 g of CaCl2 (M=111 g mol-1) was dissolved in 100 g of water, assuming CaCl2 undergoes complete ionization.(Kf for water= 1.86 K kg mol-1)
21. Write the name of monomers and their structures in the following:
i. Buna-S                                             ii. Terylene                                      iii. Nylon-6
22. An element crystallizes in a b.c.c lattice with cell edge of 400 pm. Calculate the density if 250 g of this element contain 2.5 x 1024 atoms ?
23. Due to hectic and busy schedule, Mr.Singh started taking junk food in the lunch break and slowly became habitual of eating food irregularly to excel in his field. One day during meeting he felt severe chest pain and fell down. Mr. Khanna, a close friend of Mr.Singh, took him to doctor immediately. The doctor diagnosed that Mr.Singh was suffering from acidity and prescribed some medicines. Mr.Khanna advised him to eat home made food and change his lifestyle by doing Yoga, meditation and some physical exercise. Mr.Singh followed his friend’s advice and after few days he started feeling better. After reading the above passage, answer the following:
i. What are the values (at least two) displayed by Mr.Khanna?
ii. What are antacids? Give one example.
iii. Would it be advisable to take antacids for a long period of time? Give reason.
24. a. Account for the following:
i. Mn2O7 is acidic whereas MnO is basic. ii. Zr and Hf exhibit similar properties.
iii. Transition metals form a large number of complex compounds.
b. Write type of the preparation of K2MnO4 from pyrolusite ore (MnO4).Write the type of magnetism shown by KMnO4 and K2MnO4
                                                                     OR
a. The elements of 3d transition series are given as: Sc Ti V Cr Mn Fe Co Ni Cu Zn
Answer the following:
i. Copper has exceptionally positive Eo (M2 + M) value Why?
ii. Which element is a strong reducing agent in +2 oxidation state and why?
iii.Zn2+ salts are colorless. Why?
b. Write the preparation of sodium dichromate from chromite ore (FeCr2O4).
25. a. Write the product(s) in each of the following reactions:

b. Write the chemical equations involved in the following reactions:
i. Reimer-Tiemann reaction ii. Friedal-Crafts alkylation of anisole.
                                                            OR
a. What happens when:
i. Phenol reacts with conc. HNO3     ii. Salicylic acid reacts with (CH3CO)2O/H+ iii. Ethyl chloride reacts with NaOCH3. Write the chemical equations involved in the above reactions.
b. Distinguish between:
i. Ethanol and Phenol  ii. Propan-2-ol and 2-methylpropan-2-ol
26. a. Calculate ΔG° and log Kc for the following reaction at 298 K:
2Al(s) + 3Cu2+(aq) → 2Al3+(aq) +3Cu(s)
Given: E°cell=2.02 V
b. Using the Eo values of A and B, predict which is better for coating the surface of iron[Eo(Fe2+/Fe)=-0.44 V] to prevent corrosion and why ?
Given: E0(A2+/A)=-2.37 V : Eo(B2+/B)=-0.14 V
                                                                    OR
a. The conductivity of 0.001 Calculate mol L-1solution of its molar conductivity CH3COOH is
 3.90 x10-5Scm-1.degree is of dissociation(a).
Given λ ° (H+)=349.6 S cm2 mol-1 and λ ° (CH3COO-)=40.9 S cm2 mol-1
b. What type of battery is dry cell? Write the overall reaction occurring in dry cell.

                                                          2. Chemistry

1. What is the maximum possible coordination number of an atom in an hcp crystal structure of anπelement? [1]
2. State the formula relating pressure of a gas with its mole fraction in a liquid solution in contact with it. [1]
3. Express the relation between the half-life period of a reactant and its initial concentration if the reaction involved is of second order. [1]
4. How are formalin and trioxane related to methanol? [1]
5. Why are primary amines higher boiling than tertiary amines? [1]
6. Show that the Heisenberg Uncertainty Principle is of negligible significance for an object of
10-6 kg mass.  (h/4Ï€ =0.528 x 10-34 kg m2s-1)                                         
                                                                     OR
On the basis of Heisenberg Uncertainty Principle show that the electron ( mass= 9 x 10-31kg cannot exist within an atomic nucleus of radius 10-15m. [2]
7. On the basis of the following values at1073K:
S2(s) + 2O2(g) ⟶2SO2(g); ∆Go = -544 kj mol-1
2Zn(s) + O2(g) ⟶2ZnO(s); ∆Go = -480 kj mol-1
2Zn(s) + S2(s) ⟶2ZnS(s); ∆Go = -293 kj mol-1
Show that the roasting of zinc sulphide to form zinc oxide is a spontaneous process. [2]
8. Write one chemical reaction each to show that (a) Tin (II) chloride is a reducing agent. (b) Chlorine gas can be obtained from bleaching powder. [2]
9. Describe the steps involved in the preparation of either potassium dichromate from sodium chromate or potassium permanganate from manganese dioxide. [2]
10. What are enantiomers and diastereomers? Differentiate between chiral and achiral molecules. [2]
11. Give an illustration of Reimer-Thiemann reaction. [2]
12. How bakelite made and what is is its major use? Why is bakelite a thermo-setting polymer'. [2]
13. (a) What is meant by linear combination of atomic orbital?
(b) Illustrate bonding and ant bonding molecular orbital based on bomb-nuclear dihydrogen molecule. [3]
                                                               OR
What kinds of molecular forces are expected to exist between the species in any three of the following pairs constituting mixtures?
(a) He and N2
(b) Cl2 and NO
(c) NH3 and CO2
(d) H2S and HBr
14. Aluminium metal forms a cubic close-packed crystal structure. Its atomic radius is 125 x 10-12m.
(a) Calculate the length of the side of the unit cell.
(b) How many such unit cells are there in 1.00 m 3 of aluminium? [3]
15. A solution is made by dissolving 30 g of a non-volatile solute in 90 g of water. It has a
vapor pressure of 2.8 kPa at 298 K. At 298 K, vapor pressure of pure water is 3.64 kPa.
Calculate the molar mass of the solute. [3]
16. Comment on the validity of the following statements, giving reasons: [3]
(a) Thermodynamically an exothermic reaction is sometimes not spontaneous.
(b) The entropy of steam is more than that of waterat its boiling point.
(c) The equilibrium constant for a reaction is one or more if ∆Go for it is less than zero.
17. A first order reaction takes 69.3 minutes for 50% completion. Set up an equation for
determining the time needed for 80% completion of this reaction. (Calculation of result is not
required). [3]
18. Illustrate with examples: [3]
(a) Lyophilic and Lyophobic sols
(b) Multimolecular and Macromolecular colloids
(c) Homogeneous and Heterogeneous catalysis.
19. The Evalues in respect of electrodes of chromium (Z = 24), manganese (Z = 25) and
iron (Z =26) are: [3]
Cr3+/Cr2+ = -0.4V; Mn3+/Mn2+ = +1.5v, Fe3+ = +0.8V
On the basis of the above information compare the feasibility of further oxidation of their
+2 oxidation states.

20. Draw a sketch to show the splitting of orbital inan octahedral crystal field. State for a d6
ion how the actual configuration of the split d-orbital in an octahedral crystal field is decided
by the relative values △u of and P. [3]
21. (a) Write the structural formula of 3-phenylprop-2-enal.
(b) Write one chemical equation each to illustrate the following reactions:
(i) Aldol condensation (ii) Cannizzaro's reaction [3]
22. (a) Assign a reason for each of the following statements:
(i) Alkylamines are stronger bases than arylamines.
(ii) Acetonitrile Is preferred as solvent for carrying out several organic reactions.
(b) How would you convert methylamine into ethylamine? [3]
23. When the nuclides 27/13Al, 24/12Mg, and 27/14 Si are separately subjected to [∝,n ] nuclear reactions, three separate new nuclides are produced, each of which further undergoes one
positron emission finally giving stable nuclei. Write the nuclear equations for the reactions
involved in these cases. [3]
24. (a) State the function along with one example each of: [3]
(i) Antihistamines
(ii) Antioxidants
(b) What are hybrid propellants?
25. (a) Define electrical conductivity and molar conductivity of a solution and write the units
of molar conductivity.
(b) The Evalues corresponding to the following two reduction electrode processes are:
Cu+/Cu = +0.52V
Cu2+/Cu+ = 0.16V
Formulate the galvanic cell for their combination. What will be the standard cell potential
for it? Calculate ∆Go   for the cell reaction. [5]
                                                                    Or
(a) In the button cell, widely used in watches and other devices, the following reaction takes
place:
Zn(s) + Ag2O(s) + H2O(f) ⟶Zn2+(aq) + 2Ag(s) + 2OH- (aq)
Determine Eo and  Î”G°for the reaction.
Given  Eo Ag+ / Ag = +0.80V,  Eo Zn+ / Zn = -0.76V
(b) Explain with examples the terms weak and strong electrolytes. How can these be
distinguished? [5]
26. (a) Assign an appropriate reason for each of the following statements:
(i) SiF2-6is known but SiCl2-6is not known.
(ii) More metal fluorides are ionic in nature than metal chlorides.
(iii) Solid phosphorus pentachloride exhibits some ionic character.
(b) Write the structural formulae for the following: [3, 2]
(i) BrF3
(ii) XeOF4
                                                              Or
(a) Assign a reason for each of the following:
(i) In group 14 the tendency for catenation decreases with increasing atomic numbers.
(ii) In group 15 the bond angle H - M - H decreases in the following order
NH3(107.8o ), PH3(93.6o ),AsH3(91.8o )
(iii) Sulphur hexafluoride is used as a gaseous electrical insulator.
(b) Complete the following reaction equations: [3, 2]
(i)R2SiCl2 + H2O
(ii)XeF4 + H2O
27. (a) Write chemical equations for the reactions of glucose with
(i) acetic anhydride and
(ii) ammoniacal silver nitrate solution.
(b) Draw simple Fischer projections of D-glucose and L-glucose.
(c) What do you understand by replication by DNA? How does DNA differ from RNA
structurally? [2, 1, 2]
                                                               Or
(a) Write the following about protein synthesis:
(i) Name the location where protein synthesis occurs.
(ii) How do 64 codons code for only 20 amino acids?
(ii) Which of the two bases of the codon are most important for coding?
(b) What deficiency diseases are caused due to lack of vitamins A, B, B6 and K in human diet?[3, 2]


                                                                    3. Chemistry

1. What is the reason for the stability of colloidal solution?
2. Give an example each of a molecular solid and an ionic solid.
3. Pb(NO3)2 on heating gives a brown gas which undergoes dimerization on cooling?Identify the gas.
4. Write the structure of an isomer of compound C4H9Br which is most reactive towards SN1
reaction.
5. Write the IUPAC name of the given compound:
6. When a coordination compound Co is mixed with AgNO3,3 moles of AgCl are precipitated per mole of the compound. Write i. Structural formula of the complex ii. IUPAC name of the complex
7. i. Gas (A) is more soluble in water than Gas (B) at the same temperature. Which one of the two gases will have the higher value of KH (Henry’s constant) and why? ii. In non-ideal solution, what type of deviation shows the formation of maximum boiling azeotropes?
Write the structures of the following:
(i) BrF3             (ii) XeF4
8. What happens when: i. SO2 gas is passed through an aqueous solution salt? ii. XeF4 reacts with SbF5 ?
9. Write the chemical equations involved in the following reactions:
i. Hoffmann-bromamide degradation reaction ii. Carbylamine reaction
10. For a reaction: H2 + Cl2 ➝2HCl    Rate=k
i. Write the order and molecularity of this reaction. ii. Write the unit of k.
11. a. For the complex [Fe(H2O)6]3+ , write the hybridization, magnetic character and spin of
the complex. (At. number: Fe=26) b. Draw one of the geometrical isomers of the complex [Pt(en)2Cl2]2+  which is optically inactive.
12. i. Write the structural difference between starch and cellulose.
ii. What type of linkage is present in Nucleic acids?
iii. Give one example each for fibrous protein and globular protein.
13. i. What is the role of Sulphur in the vulcanization of rubber?
ii. Identify the monomers in the following polymer:
iii. Arrange the following polymers in the increasing order of their intermolecular forces:
Terylene, Polythene, Neoprene
14. How do you convert: Write the major product(s) in the following:
i. Chlorobenzene to biphenyl ii. Propene to 1-iodopropane iii. 2-bromobutane to but-2-ene
15. Give reasons for the following:
i. Aniline does not undergo Friedal-Crafts reaction.
ii. (CH3)2 NH is more basic than (CH3)3 N in an aqueous solution.
iii. Primary amines have higher boiling point than tertiary amines.
17. Give reasons:
i. SO2 is reducing while TeO2 is an oxidizing agent.
ii. Nitrogen does not form pentahalide.
iii. ICl is more reactive than I2.
18. Calculate the boiling point of solution when 4 g of MgSO4 (M=120 g mol-1) was dissolved
in 100 g of water, assuming MgSO4  undergoes complete ionization.( for water50.52 K kg mol-1)
19. i. Name the method of refining of nickel
ii. What is the role of cryolite in the extraction of aluminium?
iii. What is the role of limestone in the extraction of iron from its oxides?
20. Define the following terms: i. Lyophilic colloid ii. Zeta potential iii. Associated colloids
21. For the first order thermal decomposition reaction, the following data were obtained:
C2H5Cl(g)→C2H4(g)+HCl(g)
Time/sec                                                       Total pressure / atm
0     0.30                                                          300              0.50
Calculate the rate constant
(Given: log 2=0.301, log 3=0.4771, log 4=0.6021)
22. An element crystallizes in a b.c.c. lattice with cell edge of 500pm. The density of the
element is 7.5g cm-3. How many atoms are present in 300 g of the element?
23. Due to hectic and busy schedule, Mr. Singh started taking junk food in the lunch break
and slowly became habitual of eating food irregularly to excel in his field. One day during
meeting he felt severe chest pain and fell down. Mr. Khanna, a close friend of Mr. Singh.
took him to doctor immediately. The doctor diagnosed that Mr. Singh was suffering from
acidity and prescribed some medicines. Mr. Khanna advised him to eat homemade food
and change his lifestyle by doing yoga, meditation and some physical exercise. Mr. Singh followed his friend’s advice and after few days he started feeling better. After reading the above passage, answer the following:
i. What are the values (at least two) displayed by Mr. Khanna?
ii. What are antacids? Give one example.
iii. Would it be advisable to take antacids for a long period of time? Give reason.
24. Account for the following:
1. Mn shows the highest oxidation state of +7 with oxygen but with fluorine it shows
the highest oxidation state of +4.
2. Cr2+ is a strong reducing agent.
3. Cr2+  salts are coloured while Zn2+  salts are white.
                                                 OR
a. The elements of 3d transition series are given as:
Sc Ti V Cr Mn Fe Co Ni Cu Zn
b. Answer the following:
i. Write the element which shows maximum number of oxidation states.Give reason.
ii. Which element has the highest m.p?
iii. Which element shows only +3 oxidation state?
iv. Which element is a strong oxidizing agent in +3 oxidation state and why?
25. a. Write the structures of A and B in the following reactions:
i. CH3COCl
ii. CH3MgBr
b. Distinguish between:
i. C6H5-COOH3  and C6H5-CHO
ii. CH3COOH and HCOOH
c. Arrange the following in the increasing order of their boiling points:
CH3CHO, CH3COOH, CH3CH2OH
                                             OR
a. Write the chemical reaction involved in Wolff-Kishner reduction.
b. Arrange the following in the increasing order of their reactivity towards nucleophilic
addition reaction:
C6H5COCH3, CH3-CHO, CH3COCH3
c. Why carboxylic acid does not give reactions of carbonyl group?
d. Write the product in the following reaction CH3CH2CH=CH2CH
e. A and B are two functional isomers of compoundC3H6O. On heating with NaOH and I2, isomer B forms yellow precipitate of iodoform whereas isomer A does not form any precipitate. Write the formulae of A and B.
26. a. Calculate E°cell for the following reaction at 298K:
 2Al(s)1+3Cu2+ (0.01M) →2A]3+(0.01M)+  3Cu(s) Given: Ecell =1.98 V
b. Using the E° values of A and B, predict which is better for coating the surface of iron[E°(Fe2+ /Fe) =-0.44V] to prevent corrosion and why? Given: E° (A2+ /A) =-2.37V: E° B2+ /B) =-0.14V
                                                                 OR
a. The conductivity of 0.001 mol L-1solution of CH3COOH is 3.905x10-5S cm-1calculateits
molar conductivity and degree of dissociation ().Given (H+ ) =349.6 S mol-1 and (CH3COO-)=40.9 S mol-1
b. Define electro chemical cell. What happens if external potential applied becomes greater than E°cell of electro chemical cell?

                                                        4. CHEMISTRY

1. On heating Zn granules with conc. HNO3, a brown gas is evolved which undergoes dimerization on cooling. Identify the gas.
2. What is the reason for the stability of colloidal sols?
3. Write the IUPAC name of the given compound:

4. Out of CH2=CH CH2Cl and CH3CH2-CH2Cl, which is more reactive towards SN1 reaction?
5. What type of magnetism is shown by a substance if its domains are arranged in equal number and in opposite directions?
6. When a coordination compound PdCl24NH3is mixed with AgNO3, 2 moles of AgCl are
precipitated per mole of the compound. Write:
(i) Structural formula of the complex (ii) IUPAC name of the complex
7. Write the chemical equations involved in the following reactions:
(i) Clemmensen reduction (ii) Hell-Volhard Zelinsky reaction
                                                  OR
How do you convert
(i) Toluene to benzaldehyde (ii) Ethanoyl chloride to ethanal
8. For a reaction: 2NH3(g) →(g)+ 3H2(g)         Rate=k
(i) Write the order and molecularity of this reaction.(ii) Write the unit of k.
9. For the given cells:
Lead storage cell, Mercury cell, Fuel cell and Dry cell
Answer the following:
(i) Which cell is used in hearing aids?
(ii) Which cell was used in Apollo space programme?
(iii) Which cell is used in automobiles and inverters?
(iv) Which cell does not have long life?
10.When pyrolusite ore MnO2is fused with KOH in presence of air, a green coloured compound (A) is obtained which undergoes disproportionation reaction in acidic medium to give a purple coloured compound (B):
(i) Write the formulae of the compounds (A) and (B).
(ii) What happens when compound (B) is heated?
11. Calculate E0cell for the following reaction at 298K:
2Cr(s) +3Fe2+(0.01M) →2Cr3(0.01M)+3Fe(s)
Given: Ecell=261 V
12. (i) Name the method of refining Zirconium.(ii) In the extraction of Al, impure Al2O3 is dissolved   in conc. NaOH to form sodium aluminate and leaving impurities behind. What is the name of this process?
(iii) What is the function of limestone in the extraction of iron from its oxides?
13. (i) Out of silica gel and anhydrous CaCl2, which will adsorb the water vapours?
(ii) Out of H2SO4 and H3PO4, which one is more effective in causing coagulation of positively
charged sol? Give reason.
(iii) Out of sulphur sol and proteins, which one forms macromolecular colloids?
14. The rate constant for the first order decomposition of H2O2 is given by the following equation:
log k =14.2    1.0 x 10 4 K  
                            T       
Calculate Ea for this reaction and rate constant k if its half - life period be 200 mintues.(Given: R=8.314 JK-1 mole-1)
15. An element crystallizes in a f.c.c. lattice with cell edge of 400 pm. The density of theelement is 7g cm-3. How many atoms are present in 280 g of the element?
16. (a) For the complex [CoF6]3-, write the hybridization, magnetic character and spin of the complex. (At. number: Co=27)
(b) Draw one of the geometrical isomers of the complex [Co(en)2CI2]+ which is optically active.
17. (i) Write the name of two monosaccharides obtained on hydrolysis of maltose sugar.
(ii) Name the vitamin whose deficiency causes convulsions.
(iii) Write one example each for Fibrous protein and Globular protein.
18. (i) What is the role of t-butyl peroxide in the polymerization of ethene?
(ii) Identify the monomers in the following polymer:


(iii) Arrange the following polymers in the increasing order of their intermolecular forces:
PVC, Nylon-6, Buna-N
                                                                    OR
Write the mechanism of free radical polymerization of ethene.
19. Complete the following reactions:





20. Give reasons for the following:
(a) Aldehydes (R-CHO) are more reactive than ketones (R-CO-R) towards nucleophilic addition
reaction.
(b) Benzaldehyde does not undergo aldol condensation reaction.
(c) Benzoic acid does not give Friedal-Crafts reaction.
21. What happens when: (i) 2, 4, 6 - trinitrochlorobenzene is treated with warm water.
(ii) 2-chlorobutane is treated with alcoholic KOH.
(iii) ethyl chloride is treated with Na metal in presence of dry ether. Write the equation involved in the above reactions.
22. Give reasons:
(i) Mn shows the highest oxidation state of 17 with oxygen but with fluorine it shows the
highest oxidation state of 14. (ii) Zn is soft whereas Cr is hard. (iii) Eu2+ is a good reducing agent.
23. Due to hectic and busy schedule, Mr. Awasthi made his life full of tensions and anxiety.
He started taking sleeping pills to overcome the depression without consulting the doctor.Mr. Roy, a close friend of Mr. Awasthi, advised him to stop taking sleeping pills and suggested to change his lifestyle by doing Yoga, meditation and some physical exercise. Mr. Awasthi followed his friend’s advice and after few days he started feeling better. After reading the above passage, answer the following:
(i) What are the values (at least two) displayed by Mr. Roy?
(ii) Why it is not advisable to take sleeping pills without consulting doctor?
(iii) What are tanquilizers? Give two examples.
24. (a) Write the product(s) in each of the following reactions:
(i) CH2=CH-CH2OH
(ii) 2CH3CH2OH
(iii)CH3CH2-O-CH2CH3
(b) Write the mechanism of the following reaction:
                                                                         OR
(a) Write equations of the following reactions:
(i) Bromine in CS2 with phenol (ii) Treating phenol with chloroform in the presence of aq. NaOH
(iii) Anisole reacts with HI
(b) Distinguish between:
(i) Ethanol and Diethyl ether (ii) Propanol and t-butyl alcohol
25. (a) Calculate the boiling point of solution when 2g of Na2SO4(M=142 g mole-1 was dissolved in 50 g of water, assuming Na2SO4 undergoes complete ionization. (Kb for water=0.52 K kg mole-1)
(b) Define the following terms: (i) Colligative properties  (ii) Ideal solution
                                                                    OR
(a) When 2.56 g of sulphur was dissolved in 100 g of the freezing point lowered by 0.383 K.
Calculate the formula of sulphur (Sx).(Kf for CS2=3.83 K kg mol-1 Atomic mass of
Sulphur =32g mole-1)
(b) Blood cells are isotonic with 0.9% sodium chloride solution. What happens if we place blood cells in a solution containing?
(i) 1.2% sodium chloride solution
(ii) 0.4% sodium chloride solution
26. (a) Account for the following:
(i) H2Te is more acidic thanH2S.  (ii) Cl5is more covalent than Cl3
(iii) Boiling points of interhalogens are little higher as compare to pure halogens.
(b) Draw the structures of:
(i) HClO4    (ii) XeOF4
                                                              OR
(i) Arrange the following in the increasing order of their reducing character:
(ii) Out of He and Xe, which one can easily form compound and why?
(iii) Write the conditions to maximize the yield of ammonia in Haber’s process.
(iv) Write two uses of Chlorine gas.
(v) How can you detect the presence of SO2 gas?










Wednesday 9 May 2018

CLASS 12 PREVIOUS YEARS QUESTIONS OF PHYSICS


  CBSE Class-12th                           
                                                       1. PHYSICS

General Instructions :
All questions are compulsory. There are 26 questions in all.
This question paper has five sections : Section A, Section B, Section C, Section D and Section E.
Section A contains five questions of one mark each, Section B contains five questions
of two marks each, Section C contains twelve questions of three marks each, Section
D contains one value based question of four marks and Section E contains three
questions of five marks each.
There is no overall choice. However, an internal choice has been provided in one
question of two marks, one question of three marks and all the three questions of
five marks weightage. You have to attempt only one of the choices in such questions.
You may use the following values of physical constants wherever necessary :
c = 3x108 m/s
h = 6.63x10-34 Js
e = 1.6x10-19 C
Ï€o= 4x10-7 T m A-1
εo= 8.854 10-12 C2N-1M-2
1/4πεo= 9x109 N m2 C-2
Mass of electron = 9.1 x 10-311kg
Mass of neutron = 1.675 x 10-27 kg
Mass of proton = 1.673x10-27 kg
Avogadro’s number = 6.023 x 1023 per gram mole
Boltzmann constant = 1.38 x 10-23JK-1

                                                                SECTION A
1. Nichrome and copper wires of same length and same radius are connected in series.
Current I is passed through them. Which wire gets heated up more ? Justify your answer.(1)
2. Do electromagnetic waves carry energy and momentum ?(1)
3. How does the angle of minimum deviation of a glass prism vary, if the incident violet light
is replaced by red light ? Give reason.(1)
4. Name the phenomenon which shows the quantum nature of electromagnetic radiation.(1)
5. Predict the polarity of the capacitor in the situation described below : (1)

                                                           SECTION B
6. Draw the intensity pattern for single slit diffraction and double slit interference. Hence,
state two differences between interference and diffraction patterns.(2)
                                                                   OR
Unpolarised light is passed through a polaroid P1
. When this polarized beam passes through
another polaroid P2 and if the pass axis of P2 makes angle with the pass axis of P1, then write the expression for the polarised beam passing through P2 . Draw a plot showing the variation of intensity when varies from 0 to 2 .
7. Identify the electromagnetic waves whose wavelengths vary as (2)
(a) 10-12 m <λ 10-8 m
(b) 10-3 m < λ 10-1 m
Write one use for each.
8. Find the condition under which the charged particles moving with different speeds in the
presence of electric and magnetic field vectors can be used to select charged particles of a particular speed.(2)
9. A 12·5 eV electron beam is used to excite a gaseous hydrogen atom at room temperature. Determine the wavelengths and the corresponding series of the lines emitted.(2)
10. Write two properties of a material suitable for making (a) a permanent magnet, and (b)
an electromagnet.(2)
                                    SECTION C
11. (a) The potential difference applied across a given resistor is altered so that the heat produced per second increases by a factor of 9. By what factor does the applied potential difference change ?
(b) In the figure shown, an ammeter A and a resistor of 4 are connected to the terminals of the source. The emf of the source is 12 V having an internal resistance of 2 . Calculate the
voltmeter and ammeter readings.(3)

12. (a) How is amplitude modulation achieved ? (b) The frequencies of two side bands in an AM wave are 640 kHz and 660 kHz respectively. Find the frequencies of carrier and modulating signal. What is the bandwidth required for amplitude modulation ? (3)
13. (a) In the following diagram, is the junction diode forward biased or reverse biased ?

(b) Draw the circuit diagram of a full wave rectifier and state how it works.()
14. Using photon picture of light, show how Einstein’s photoelectric equation can be
established. Write two features of photoelectric effect which cannot be explained by wave theory.(3)
15. (a) Monochromatic light of wavelength 589 nm is incident from air on a water surface. If for water is 1·33, find the wavelength, frequency and speed of the refracted light.(3)
(b) A double convex lens is made of a glass of refractive index 1·55, with both faces of the same radius of curvature. Find the radius of curvature required, if the focal length is 20 cm.
16. Define mutual inductance between a pair of coils. Derive an expression for the mutual inductance of two long coaxial solenoids of same length wound one over the other.(3)
                                                                     OR
Define self-inductance of a coil. Obtain the expression for the energy stored in an inductor L onnected across a source of emf.
17. (a) Write the principle of working of a metre bridge.
(b) In a metre bridge, the balance point is found at a distance l1 with resistances R and S as shown in the figure. An unknown resistance X is now connected in parallel to the resistance S and the balance point is found at a distance l2. Obtain a formula for X in terms of l1, l2 and S.(3)

18. Draw a block diagram of a generalized communication system. Write the functions of each of the following : (3)
(a) Transmitter
(b) Channel
(c) Receiver
19. (a) Write the functions of the three segments of a transistor.
(b) The figure shows the input waveforms A and B for ‘AND’ gate.
Draw the output waveform and write the truth table for this logic gate.(3)

20. (a) Draw a ray diagram depicting the formation of the image by an astronomical
telescope in normal adjustment. (3)
(b) You are given the following three lenses. Which two lenses will you use as an eyepiece and as an objective to construct an astronomical telescope ? Give reason.
Lenses      Power (D)     Aperture (cm)
L1                          3                     8
L2                          6                     1
L3                         10                    1
21. (a) State Biot – Savart law and express this law in the vector form.(3)
(b) Two identical circular coils, P and Q each of radius R, carrying currents 1 A and
√3 A respectively, are placed concentrically and perpendicular to each other lying in the XY and YZ planes. Find the magnitude and direction of the net magnetic field at the centre of the coils.
22. Two identical parallel plate capacitors A and B are connected to a battery of V volts with the switch S closed. The switch is now opened and the free space between the plates of the capacitors is filled with a dielectric of dielectric constant K. Find the ratio of the total electrostatic energy stored in both capacitors before and after the introduction of the dielectric.(3)

                                                                     SECTION D
23. Asha’s mother read an article in the newspaper about a disaster that took place at Chernobyl. She could not understand much from the article and asked a few questions from Asha regarding the article. Asha tried to answer her mother’s questions based on what she learnt in Class XII Physics.(4)
(a) What was the installation at Chernobyl where the disaster took place ? What, according to you, was the cause of this disaster ?
(b) Explain the process of release of energy in the installation at Chernobyl.
(c) What, according to you, were the values displayed by Asha and her mother ?
                                                                       SECTION E
24. (a) Derive an expression for the electric field E due to a dipole of length ‘2a’ at a point distant r from the centre of the dipole on the axial line.
(b) Draw a graph of E versus r for r >> a.
(c) If this dipole were kept in a uniform external electric field E0, diagrammatically represent the position of the dipole in stable and unstable equilibrium and write the expressions for
the torque acting on the dipole in both the cases.(5)
(a) Use Gauss’s theorem to find the electric field due to a uniformly charged infinitely large plane thin sheet with surface charge density .
(b) An infinitely large thin plane sheet has a uniform surface charge density +. Obtain the expression for the amount of work done in bringing a point charge q from infinity to a point, distant r, in front of the charged plane sheet.(5)
25. A device ‘X’ is connected to an ac source V =L1sin t. The variation of voltage, current and power in one cycle is shown in the following graph :
(a) Identify the device ‘X’.
(b) Which of the curves A, B and C represent the voltage, current and the power consumed in the circuit ? Justify your answer.
(c) How does its impedance vary with frequency of the ac source ? Show graphically.
(d) Obtain an expression for the current in the circuit and its phase relation with ac voltage. (5)
                                                                          OR
(a) Draw a labelled diagram of an ac generator. Obtain the expression for the emf induced in
the rotating coil of N turns each of cross-sectional area A, in the presence of a magnetic field B
(b) A horizontal conducting rod 10 m long extending from east to west is falling with a speed
5·0 ms1at right angles to the horizontal component of the Earth’s magnetic field, 0·3 x 10-4Wb m-2 Find the instantaneous value of the emf induced in the rod.(5)
26. (a) Define wavefront. Use Huygens’ principle to verify the laws of refraction.
(b) How is linearly polarised light obtained by the process of scattering of light ? Find the Brewster angle for air – glass interface, when the refractive index of glass = 1·5. (5)
                                                                        OR
(a) Draw a ray diagram to show the image formation by a combination of two thin convex lenses in contact. Obtain the expression for the power of this combination in terms of thefocal lengths of the lenses.(5)
(b) A ray of light passing from air through an equilateral glass prism undergoes minimum deviation when the angle of incidence is of the angle of prism. Calculate the speed of light in the prism.     

CBSE Class 12
                                                                    2. PHYSICS
SET-2 2017
                                                                  SECTION – A
1. A point charge Q is placed at point ‘O’ as shown in the figure. Is the potential at point A, i.e. VA , greater, smaller or equal to potential, VB , at point B, when Q is
(i) positive, and (ii) negative charge ? (1)
                                                 O.           A.               B.
2. Write the expression for speed of electromagnetic waves in a medium of electrical permittivity ε and magnetic permeability μ. (1)
3. Does the magnifying power of a microscope depend on the colour of the light used ? Justify your answer. (1)
4. Draw logic symbol of an OR gate and write its truth table. (1)
5. A photosensitive surface emits photoelectrons when red light falls on it. Will the surface emit photoelectrons when blue light is incident on it ? Give reason. (1)
                                                                  SECTION B
6. Find the intensity at a point on a screen in Young’s double slit experiment where the
interfering waves of equal intensity have a path difference of (i) λ/4, and (ii) λ/3. (2)
7. Write two points of difference between intrinsic and extrinsic semiconductors. (2)
8. Distinguish between broadcast mode and point-to-point mode of communication and give one example for each. (2)
9. A light bulb and a solenoid are connected in series across an ac source of voltage. Explain, how the glow of the light bulb will be affected when an iron rod is inserted in the solenoid.(2)
10. Use the mirror equation to show that an object placed between f and 2f of a concave
mirror forms an image beyond 2f. (2)
                                                                              OR
 (a) State the condition under which a large magnification can be achieved in an
astronomical telescope.
(b) Give two reasons to explain why a reflecting telescope is preferred over a refracting telescope.
                                                                       SECTION C
11. (a) Define the term ‘modulation index,’ used in communication system. Why is its value kept less than or equal to one?
(b) A message signal of frequency 10 kHz and peak voltage of 10 V is used to modulate a carrier frequency 1 MHz and peak voltage 10 V. Determine the (i) modulation index, and (ii) side bands produced. (3)
12. Using Bohr’s postulates, derive the expression for the orbital period of the electron moving in the nth orbit of hydrogen atom. (3)
13. A charge Q is distributed uniformly over a metallic sphere of radius R. Obtain the
expressions for the electric field (E) and electric potential (V) at a point 0 < x < R. Show on a plot the variation of E and V with x for 0 < x < 2R. (3)
14. In the given circuit, with steady current, calculate the potential difference across the
capacitor and the charge stored in it. (3)

15. A long charged cylinder of linear charge density + λ1 is surrounded by a hollow coaxial conducting cylinder of linear charge density -λ2 . Use Gauss’s law to obtain expressions for
the electric field at a point (i) in the space between the cylinders, and (ii) outside the larger cylinder.
16. Using Biot-Savart law, deduce the expression for the magnetic field at a point (x) on the axis of a circular current carrying loop of radius R. How is the direction of the magnetic field determined at this point ? (3)

                                                                          OR
The figure shows three infinitely long straight parallel current carrying conductors. Find the
(i) magnitude and direction of the net magnetic field at point A lying on conductor 1, (ii) magnetic force on conductor 2.
17. (a) State the law of radioactive decay. Write the SI unit of ‘activity’.
(b) There are radioactive nuclei in a given radioactive sample. If the half-life of the sample is 20 s, how many nuclei will decay in 10 s ? (3)
18. (a) How are electromagnetic waves produced ? Explain.
(b) A plane electromagnetic wave is travelling through a medium along the +ve z-direction. Depict the electromagnetic wave showing the directions of the oscillating electric and magnetic fields. (3)
19. A source of ac voltage v = v0 sin ωt, is connected across a pure inductor of inductance L.
Derive the expressions for the instantaneous current in the circuit. Show that average power dissipated in the circuit is zero. (3)
20. (a) Draw a plot showing the variation of photoelectric current with collector potential for different frequencies but same intensity of incident radiation.
(b) Use Einstein’s photoelectric equation to explain the observations from this graph.
(c) What change will you observe if intensity of incident radiation is changed but the frequency remains the same? (3)
21. (a) State the condition under which a charged particle moving with velocity v goes undeflected in a magnetic field B.
(b) An electron, after being accelerated through a potential difference of 10 4 V, enters a uniform magnetic field of 0·04 T, perpendicular to its direction of motion. Calculate the radius of curvature of its trajectory. (3)
22. A monochromatic light of wavelength λ is incident normally on a narrow slit of width ‘a’ to produce a diffraction pattern on the screen placed at a distance D from the slit. With the help of a relevant diagram, deduce the conditions for obtaining maxima and minima on the screen. Use these conditions to show that angular width of central maximum is twice the angular width of secondary maximum. (3)
                                                                   SECTION D
23. Sunil and his parents were travelling to their village in their car. On the way his mother noticed some grey coloured panels installed on the roof of a low building. She enquired from Sunil what those panels were and Sunil told his mother that those were solar panels.
(a) What were the values displayed by Sunil and his mother ? State one value for each.
(b) In what way would the use of solar panels prove to be very useful?
(c) Name the semiconductor device used in solar panels. Briefly explain with the help of a diagram, how this device works. (4)
                                                                   SECTION E
24. (a) (i) State the principle on which a potentiometer works. How can a given potentiometer be made more sensitive ?
(ii) In the graph shown below for two potentiometers, state with reason which of the two potentiometers, A or B, is more sensitive.
(b) Two metallic wires, P1 and P2 of the same material and same length but different cross￾sectional areas, A1 and A2 are joined together and connected to a source of emf. Find the ratio of the drift velocities of free electrons in the two wires when they are connected (i) in series, and (ii) in parallel.

                                                                            OR
(a) Define the capacitance of a capacitor. Obtain the expression for the capacitance of a parallel plate capacitor in vacuum in terms of plate area A and separation d between the plates.
(b) A slab of material of dielectric constant K has the same area as the plates of a parallel plate capacitor but has a thickness . Find the ratio of the capacitance with dielectric inside it to its capacitance without the dielectric.
25. (a) State Faraday’s law of electromagnetic induction.
(b) The magnetic field through a circular loop of wire 12 cm in radius and 8·5 resistance, changes with time as shown in the figure. The magnetic field is perpendicular to the plane of the loop. Calculate the induced current in the loop and plot it as a function of time.
(c) Show that Lenz’s law is a consequence of conservation of energy.

                                                                          OR
(a) Describe, with the help of a suitable diagram, the working principle of a step-up
transformer. Obtain the relation between input and output voltages in terms of the number of turns of primary and secondary windings and the currents in the input and output circuits.
(b) Given the input current 15 A and the input voltage of 100 V for a step-up transformer having 90% efficiency, find the output power and the voltage in the secondary if the output current is 3 A.
26. (a) A point object is placed on the principal axis of a convex spherical surface of radius of curvature R, which separates the two media of refractive indices n1 and n2 (n2 > n1). Draw the ray diagram and deduce the relation between the object distance (u), image distance (v) and the radius of curvature (R) for refraction to take place at the convex spherical surface from rarer to denser medium.
(b) A converging lens has a focal length of 20 cm in air. It is made of a material of refractive index 1·6. If it is immersed in a liquid of refractive index 1·3, find its new focal length.
                                                                       OR
 (a) Draw the ray diagram showing refraction of light through a glass prism and hence obtain the relation between the refractive index μ of the prism, angle of prism and angle of minimum deviation.
(b) Determine the value of the angle of incidence for a ray of light travelling from a medium of refractive index into the medium of refractive index μ2 = 1, so that it just grazes along the surface of separation.

CBSE Class 12 th Physics                          3. PHYSICS
                                                           
                                                                     SECTION – A
1. A point charge +Q is placed in the vicinity of a conducting surface. Draw the electric field lines between the surface and the charge. (1)
2. Define modulation index. Why is it generally kept less than one? (1)
3. In the figure given, mark the polarity of plates A and B of a capacitor when the magnets are quickly moved towards the coil. (1)
                                    
4. The objective lenses of two telescopes have the same apertures but their focal lengths are in the ratio 1: 2. Compare the resolving powers of the two telescopes.
5. Define the conductivity of a conductor. Write its SI unit. (1)
                                                                       SECTION – B
6. (i) Define refractive index of a medium.
(ii) In the following ray diagram, calculate the speed of light in the liquid of unknown
refractive index. (2)

7. Electrons are emitted from the cathode of a photocell of negligible work function, when photons of wavelength are incident on it. Derive the expression for the de Broglie wavelength of the electrons emitted in terms of the wavelength of the incident light. (2)
                                                                              OR
Derive the Bohr’s quantisation condition for angular momentum of the orbiting of electron in hydrogen atom, using de Broglie’s hypothesis.
8. (a) Write two characteristic features of nuclear force.
(b) Draw a plot of potential energy of a pair of nucleons as a function of their separation. (2)
9. State the two points to distinguish between sky wave and space wave modes of propagation. (2)
10. The figure shows a plot of terminal voltage ‘V’ versus the current ‘i’ of a given cell. Calculate from the graph (a) emf of the cell and (b) internal resistance of the cell.(2)
   
                                                            SECTION – C
11. A parallel plate capacitor of capacitance C is charged to a potential V by a battery.
Without disconnecting the battery, the distance between the plates is tripled and a dielectric medium of k = 10 is introduced between the plates of the capacitor. Explain giving reasons, how will the following be affected: (3)
(i) capacitance of the capacitor (ii) charge on the capacitor, and (iii) energy density of the capacitor.
12. (a) Draw a graph showing the variation of binding energy per nucleon (BE/A) vs mass number A for the nuclei in
 20 ≤ A ≤ 170.
(b) A nucleus of mass number 240 and having binding energy/nucleon 7.6 MeV splits into two fragments Y, Z of mass numbers 110 and 130 respectively. If the binding energy/nucleon of Y, Z is equal to 8.5 MeV each, calculate the energy released in the nuclear reaction. (3)
13. (a) In Young’s double slit experiment, the two slits are illuminated by two different lamps having same wavelength of light. Explain with reason, whether interference pattern will be observed on the screen or not.
(b) Light waves from two coherent sources arrive at two points on a screen with path differences of 0 and /2. Find the ratio of intensities at the points. (3)
14. Using Bohr’s postulates, derive the expression for the total energy of the electron revolving in nth orbit of hydrogen atom. Find the wavelength of H line, given the value of Rydberg constant,
R = 1.1× 107 m–1. (3)
15. Name the e.m. waves in the wavelength range 10 nm to 10–3 nm. How are these waves generated ? Write their two uses. (3)
16. (a) Draw the pattern of magnetic field lines for a circular coil carrying current.
(b) Two identical circular loops X and Y of radius R and carrying the same current are kept in perpendicular planes such that they have a common centre at P as shown in the figure. Find the magnitude and direction of the net magnetic field at the point P due to the loops. (3)

17. State the reason, why the photodiode is always operated under reverse bias. Write the working principle of operation of a photodiode. The semiconducting material used to fabricate a photodiode, has an energy gap of 1.2eV. Using calculations, show whether it can detect light of wavelength of 400 nm incident on it. (3)
18. Draw the circuit diagram of a common emitter transistor amplifier. Write the expression for its voltage gain. Explain, how the input and output signals differ in phase by 180°. (3)
                                                                     OR
Draw the circuit diagram of a full wave rectifier. Explain its working principle. Draw the input and output waveforms.
19. Briefly explain the three factors which justify the need of modulating low frequency signal into high frequencies. (3)
20. Define the term current sensitivity of a galvanometer. In the circuits shown in the figures, the galvanometer shows no deflection in each case. Find the ratio of R1 and R2 . (3)

21. The current through two inductors of self-inductance 12 mH and 30 mH is increasing with time at the same rate. Draw graphs showing the variation of the
(a) emf induced with the rate of change of current in each inductor
(b) energy stored in each inductor with the current flowing through it. Compare the energy stored in the coils, if the power dissipated in the coils is the same. (3)
22. (a) Explain how the intensity of diffraction pattern changes as the order (n) of the diffraction band varies.
(b) Two wavelengths of sodium light 590 nm and 596 nm are used in turn to study the diffraction at a single slit of size 4 mm. The distance between the slit and screen is 2 m.
Calculate the separation between the positions of the first maximum of the diffraction pattern obtained in the two cases. (3)
                                                                  SECTION – D
23. Mr Kamath, the chief mechanical engineer, in Northern railways went to Tokyo to attend a seminar on fast moving trains. His friend Mr Hiorki explained how Japanese people are concentrating on energy conservation and saving fossil fuels using maglev trains. Mr Kamath travelled from Tokyo to Osaka in maglev train and found that the sound is less,L travel is smooth and understood the Japanese technology in mass transporting systems. Maglev trains work on the principle of Meissner’s effect. (3+2)
(a) Mention two values which Mr Kamath found in Mr Hiorki.
(b) Which values in Mr Kamath do you appreciate ?
(c) What is Meissner’s effect ? Write the value of magnetic permeability for perfect
diamagnetism.
                                                                     SECTION – E
24. (a) State Gauss’ law. Using this law, obtain the expression for the electric field due to an infinitely long straight conductor of linear charge density λ.
(b) A wire AB of length L has linear charge density λ = kx, where x is measured from the end A of the wire. This wire is enclosed by a Gaussian hollow surface. Find the expression for the electric flux through this surface. (3+2)
                                                                          OR
 (a) Derive the expression for the electric potential at any point P, at distance r from the centre of an electric dipole, making angle α, with its axis.
(b) Two point charges 4 μC and +1 μC are separated by a distance of 2 m in air. Find the point on the line-joining charges at which the net electric field of the system is zero.
25. (a) Prove that an ideal capacitor in an ac circuit does not dissipate power.
(b) An inductor of 200 mH, capacitor of 400 μf and a resistor of 10 are connected in series to ac source of 50 V of variable frequency. Calculate the (i) angular frequency at which maximum power dissipation occurs in the circuit and the corresponding value of the effective current, and (ii) value of Q-factor in the circuit. (3+2)
                                                                         OR
(a) A metallic rod of length l is moved perpendicular to its length with velocity ν in a magnetic field acting perpendicular to the plane in which rod moves. Derive the expression for the induced emf.
(b) A wheel with 15 metallic spokes each 60 cm long, is rotated at 360 rev/min in a plane normal to the horizontal component of earth’s magnetic field. The angle of dip at that place is 60°. If the emf induced between rim of the wheel and the axle is 400 mV, calculate the horizontal component of earth’s magnetic field at the place. How will the induced emf change, if the number of spokes is increased?
26. (a) Explain with reason, how the power of a diverging lens changes when (i) it is kept in a medium of refractive index greater than that of the lens. (ii) incident red light is replaced by violet light.
(b) Three lenses L1, L2, L3 each of focal length 30 cm are placed co-axially as shown in the figure. An object is held at 60 cm from the optic centre of lens L1 . The final real image is formed at the focus of L3 . Calculate the separation between (i) (L1 and L2) and (ii) (L2 and L3). (3+2)

                                                                         OR
(a) Deduce the expression, by drawing a suitable ray diagram, for the refractive index of a triangular glass prism in terms of the angle of minimum deviation (D) and the angle of prism (A). Draw a plot showing the variation of the angle of deviation with the angle of incidence.
(b) Calculate the value of the angle of incidence when a ray of light incident on one face of an equilateral glass prism produces the emergent ray, which just grazes along the adjacent face. Refractive index of the prism is √2.

CBSE Class 12
th Physics
Last Year Paper
Compartment Delhi SET-1 2017                    4. PHYSICS

                                                              SECTION – A
1. Draw a plot showing variation of electric field with distance from the centre of a solid conducting sphere of radius R, having a charge of +Q on its surface. (1)
2. State one factor which determines the intensity of light in the photon picture of light. (1)
3. An iron-cored solenoid has self-inductance 2.8 H. When the core is removed, the self-inductance become 2 mH. What is the relative permeability of the core used? (1)
4. An object is kept in front of a concave lens. What is the nature of the image formed ?
5. When light travels from a rarer medium to denser medium, the speed of light decreases.
Does the reduction in speed imply a reduction in the energy? (1)

                                                              SECTION – B
6. How is electromagnetic wave produced ? Draw a sketch of a plane e.m. wave propagating
along X-axis depicting the directions of the oscillating electric and magnetic fields. (2)
7. A charge q of mass m is moving with a velocity of V, at right angles to a uniform magnetic field B. Deduce the expression for the radius of the circular path it describes.
8. Calculate the shortest wavelength of light emitted in the Paschen series of hydrogen
spectrum. Which part of the electromagnetic spectrum, does it belong ? Given :
Rydberg constant, R = 1.1 × 10 7 m-1 . (2)
9. A small illuminated bulb is at the bottom of a tank, containing a liquid of refractive index upto a height H. Find the expression for the diameter of an opaque disc, floating
symmetrically on the liquid surface in order to cut-off the light from the bulb. (2)
                                                                     OR
 A ray of light is incident on a glass prism of refractive index and refracting angle A. If
it just suffers total internal reflection at the other face, obtain an expression relating the
angle of incidence, angle of prism and critical angle.
10. Depict the behaviour of magnetic field lines near (i) diamagnetic and (ii) paramagnetic
substances. Justify, giving reasons. (2)
                                                          SECTION – C
11. Draw a graph showing the variation of de Broglie wavelength of a particle of charge q
and mass m with the accelerating potential. Proton and deuteron have the same de Broglie
wavelengths. Explain which has more kinetic energy. (3)
12. Explain the term, ‘amplitude modulation’ of a signal. For an amplitude modulated wave,
the maximum amplitude is 10 V and the minimum amplitude is 2 V. Calculate the
modulation index. (3)
13. State the Lorenz’s force and express it in vector form. Which pair of vectors are always
perpendicular to each other? Derive the expression for the force acting on a current carrying
conductor of length L in a uniform magnetic field ‘B’. (3)
14. An optical instrument uses eye-lens of power 16 D and objective lens of power 50 D and has a tube length of 16.25 cm. Name the optical instrument and calculate its magnifying power if it forms the final image at infinity.
15. Explain the two processes involved in the formation of a p-n junction diode. Hence define the term ‘barrier potential’.
16. (a) Write two properties by which electric potential is related to the electric field.
(b) Two point charges q1 and q2, separated by a distance of r12 are kept in an externalelectric field. Derive an expression for potential energy of the system of two charges in field.
                                                                     OR
State Gauss’s law in electrostatics. Derive an expression for the electric field due to an
infinitely long straight uniformly charged wire. (3)
17. State Lenz’s law. Explain, by giving examples that Lenz’s law is a consequence of conservation of energy. (3)
18. A capacitor of unknown capacitance is connected across a battery of V volt. A charge of 360 μC is stored in it. When the potential across the capacitor is reduced by 120 V, the charge stored in the capacitor becomes 120 μC. Calculate V and the unknown capacitance. What would have been the charge on capacitor if the voltage were increased by 120 V? (3)
19. A plane wavefront propagating from a rarer into a denser medium is incident at an angle of incidence i on a refracting surface. Draw a diagram showing incident wavefront and refracted wavefront. Hence verify Snell’s laws of refraction. (3)
20. Distinguish between sky wave and space wave modes of communication. What is the
main limitation of space wave mode ? Write the expression for the optimum separation between the transmitting and receiving antenna for effective reception of signals in this mode of communication. (3)
21. Using the wave forms of the input A and B, draw the output waveform of the given logic circuit. Identify the logic gate obtained. Write also the truth table. (3)
22. Derive the expression for the current density of a conductor in terms of the conductivity and applied electric field. Explain, with reason how the mobility of electrons in a conductor changes when the potential difference applied is doubled, keeping the temperature of the conductor constant. (3)

                                                                 SECTION – D
23. Ram was a daily wage worker in a factory. He was suffering from Cancer. On hearing this, most of his co-workers, started avoiding him under the impression that it was a contagious disease. When Prof. Srivatsava came to know about this case, he took him to a leading radiologist, who examined him and told that it was at the beginning stage. He advised that it could be easily cured and also certified that it was not a communicable disease. After this, Ram was given proper treatment by the doctor and got cured completely.
(1) What moral values did Prof. Srivatsava display ?
(2) How is mean life of a radioactive element related to its half life ? (4)
(3) A radioactive sample has activity of 10,000 disintegrations per second after 20 hours. After next 10 hours its activity reduces to 5,000 dps. Find out its halflife and initial activity.

                                                                  SECTION – E
24. In the following circuit, calculate (a) the capacitance of the capacitor, if the power factor of the cir cuit is unity, (b) the Q-factor of this circuit. What is the significance of the Q-factor in a.c. circuit ? Given the angular frequency of the a.c. source to be 100/s. Calculate the average power dissipated in the circuit.
OR (a) Prove that the current flowing through an ideal inductor connected across a.c. source, lags the voltage in phase by 2.
(b) An inductor of self-inductance 100 mH, and a bulb are connected in series with a.c.
source of rms voltage 10 V, 50 Hz. It is found that effective voltage of the circuit leads the
current in phase by . Calculate the inductance of the inductor used and average power dissipated in the circuit, if a current of 1 A flows in the circuit. (5)

25. Explain with diagram, how plane polarized light can be produced by scattering of
sunlight. An incident beam of light of intensity Io is made to fall on a polaroid A. Another polaroid B is so oriented with respect to A that there is no light emerging out of B. A third polaroid C is now introduced mid-way between A and B and is so oriented that its axis bisects the angle between the axes of A and B. Calculate the intensity of light transmitted by A, B andC.
                                                                         OR
(a) In Young’s double slit experiment, a monochromatic source of light S is kept
equidistant from the slits S1 and S2
. Explain the formation of dark and bright fringes on the screen.
(b) A beam of light consisting of two wavelengths, 650 nm and 520 nm, is used to obtain interference fringes in a Young’s double-slit experiment.
(i) Find the distance of the third bright fringe on the screen from the central maximum for wavelength 650 nm.
(ii) What is the least distance from the central maximum where the bright fringes due to both the wavelengths coincide?
Given: the separation between the slits is 4 mm and the distance between the screen and plane of the slits is 1.2 m. (5)
26. (a) Draw a circuit diagram of a meter bridge used to determine the unknown resistance R of a given wire. Hence derive the expression for R in terms of the known resistance S.
(b) What does the term ‘end error’ in a metre bridge circuit mean and how is it corrected ? How will the balancing point be affected, if the positions of the battery and galvanometer are interchanged in a metre bridge experiment ? Give reason for your answer.
                                                                               OR
(a) State the working principle of a potentiometer with help of the circuit diagram, explain how the internal resistance of a cell is determined.
(b) How are the following affected in the potentiometer circuit when (i) the internal
resistance of the driver cell increases and (ii) the series resistor connected to the driver cell is reduced? Justify your answer. (5)

                                                                      5. PHYSICS

                                                               SECTION A
Question Paper 2016 Central Outside Delhi Set 1

1. In what way is the behaviour of a diamagnetic material different from that of a paramagnetic, when kept in an external magnetic field?
2. The plot of the variation of potential difference across a combination of three identical cells in series, versus current is shown below. What is the emf and internal resistance of each cell?


3. Why does sun appear red at sunrise and sunset?
4. A charge ‘q’ is moved from a point A above a dipole of dipole moment ‘p’ to a point B below the dipole in equatorial plane without acceleration. Find the work done in the process.


5. Name the essential components of a communication system.

                                                              SECTION B
6. Calculate the de-Broglie wavelength of the electron orbiting in the n = 2 state of hydrogen
atom.
7. A battery of emf 12V and internal resistance 2 V is connected to a 4 V resistor as shown in
the figure.
(a) Show that a voltmeter when placed across the cell and across the resistor, in turn, gives
the same reading.
(b) To record the voltage and the current in the circuit, why is voltmeter placed in parallel and ammeter in series in the circuit?

8. Define ionization energy.
How would the ionization energy change when electron in hydrogen atom is replaced by a particle of mass 200 times that of the electron but having the same charge
                                                                      OR
Calculate the shortest wavelength of the spectral lines emitted in Balmer series.
[Given Rydberg constant, R = 107 M-1]
9. Define modulation index. Why is it kept low? What is the role of a bandpass filter?
10. A ray PQ incident normally on the refracting face BA is refracted in the prism BAC made of material of refractive index 1.5. Complete the path of ray through the prism. From which face will the ray emerge? Justify your answer.

 
                                             SECTION C
11. (i) Derive an expression for drift velocity of free electrons.
(ii) How does drift velocity of electrons in a metallic conductor vary with increase in
temperature ? Explain.
12. (a) Write the basic nuclear process involved in the emission of in a symbolic form, by a radioactive nucleus. (b) In the reactions given below :
(i)11C6→zBy + x + v
(ii) 12C6 + 12C6→20 Ne + c He b
Find the values of x, y, andz and a, b and c.
13. Sketch the graphs showing variation of stopping potential with frequency of incident radiations for two photosensitive materials A and B having threshold frequencies vA > vB.
(i) In which case is the stopping potential more and why ?
(ii) Does the slope of the graph depend on the nature of the material used ? Explain.
14. (i) State law of Malus.
(ii) Draw a graph showing the variation of intensity (I) of polarised light transmitted by an analyser with angle (u) between polariser and analyser.
(iii) What is the value of refractive index of a medium of polarising angle 60°?
15. Define an equipotential surface. Draw equipotential surfaces :
(i) in the case of a single point charge and
(ii) in a constant electric field in Z-direction.
Why the equipotential surfaces about a single charge are not equidistant ?
(iii) Can electric field exist tangential to an equipotential surface ? Give reason.
16. Use Biot-Savart law to derive the expression for the magnetic field on the axis of a current carrying circular loop of radius R.
Draw the magnetic field lines due to a circular wire carrying current I.
17. Define the term wave front. State Huygen’s principle.
Consider a plane wave front incident on a thin convex lens. Draw a proper diagram to show how the incident wave front traverses through the lens and after refraction focusses on the focal point of the lens, giving the shape of the emergent wave front.
                                                                          OR
Explain the following, giving reasons:
(i) When monochromatic light is incident on a surface separating two media, the reflected and refracted light both have the same frequency as the incident frequency.
(ii) When light travels from a rarer to a denser medium, the speed decreases. Does this decrease in speed imply a reduction in the energy carried by the wave ?
(iii) In the wave picture of light, intensity of light is determined by the square of the
amplitude of the wave. What determines the intensity in the photon picture of light ?
18. For a CE-transistor amplifier, the audio signal voltage across the collector resistance of 2 kV is 2 V. Suppose the current amplification factor of the transistor is 100, find the input
signal voltage and base current, if the base resistance is 1 kV.
19. (i) Identify the part of the electromagnetic spectrum which is :
(a) suitable for radar system used in aircraft navigation,
(b) produced by bombarding a metal target by high speed electrons.
(ii) Why does a galvanometer show a momentary deflection at the time of charging or discharging a capacitor ? Write the necessary expression to explain this observation.
20. (i) Which mode of propagation is used by shortwave broadcast services having frequency range from a few MHz upto 30 MHz ? Explain diagrammatically how long distance communication can be achieved by this mode.
(ii) Why is there an upper limit to frequency of waves used in this mode ?
21. (i) Explain with the help of a diagram the formation of depletion region and barrier potential in a pn junction.
(ii) Draw the circuit diagram of a half wave rectifier and explain its working.
22. (i) When an AC source is connected to an ideal inductor show that the average power supplied by the source over a complete cycle is zero.
(ii) A lamp is connected in series with an inductor and an AC source. What happens to the brightness of the lamp when the key is plugged in and an iron rod is inserted inside the inductor ? Explain
                                                           SECTION D
23. Ram is a student of class X in a village school. His uncle gifted him a bicycle with a
dynamo fitted in it. He was very excited to get it. While cycling during night, he could light the bulb and see the objects on the road. He, however, did not know how this device works. He asked this question to his teacher. The teacher considered it an opportunity to explain the
working to the whole class. Answer the following questions:
(a) State the principle and working of a dynamo.
(b) Write two values each displayed by Ram and his school teacher.

                                                                SECTION - E
24. (i) Derive the mathematical relation between refractive indices n1 and n2 of two radii and radius of curvature R for refraction at a convex spherical surface. Consider the object to be a point since lying on the principle axis in rarer medium of refractive index n1 and a real image formed in the denser medium of refractive index n2. Hence, derive lens maker’s
formula.
(ii) Light from a point source in air falls on a convex spherical glass surface of refractive index 1.5 and radius of curvature 20 cm. The distance of light source from the glass surface is 100 cm. At what position is the image formed?
                             OR
(a) Draw a labelled ray diagram to obtain the real image formed by an astronomical
telescope in normal adjustment position. Define its magnifying power.
(b) You are given three lenses of power 0.5 D, 4 D and 10 D to design a telescope.
(i) Which lenses should  he used as objective and eyepiece ? Justify your answer.
(ii) Why is the aperture of the objective preferred to be large?
25. (i) Use Gauss’s law to find the electric field due to a uniformly charged infinite plane sheet. What is the direction of field for positive and negative charge densities ?
(ii) Find the ratio of the potential differences that must be applied across the parallel and series combination of two capacitors C1 and C2 with their capacitances in the ratio 1 : 2 so that the energy stored in the two cases becomes the same.
                                                                         OR
(i) If two similar large plates, each of area A having surface charge densities and
are separated by a distance d in air, find the expressions for (a) field at points between the two plates and on outer side of the plates. Specify the direction of the field in each case.
(b) the potential difference between the plates.
(c) the capacitance of the capacitor so formed. (ii) Two metallic spheres of radii R and 2R are charged so that both of these have same surface charge density s. If they are connected to each other with a conducting wire, in which direction will the charge flow and why?
26. (i) Draw a labelled diagram of a step-down transformer. State the principle of its working.
(ii) Express the turn ratio in terms of voltages.
(iii) Find the ratio of primary and secondary currents in terms of turn ratio in an ideal transformer.
(iv) How much current is drawn by the primary of a transformer connected to 220 V supply when it delivers power to a 110 V-2550 W refrigerator ?
                                                                            OR
(a) Explain the meaning of the term mutual inductance. Consider two concentric circular coils, one of radius r1 and the other of radius r2(r1 < r2) placed coaxially with centres coinciding with each other. Obtain the expression for the mutual inductance of the arrangement.
(b) A rectangular coil of area A, having number of turns N is rotated at ‘f ‘ revolutions per second in a uniform magnetic field B, the field being perpendicular to the coil. Prove that the maximum emf induced in the coil is 2 NBA.